LSAT and Law School Admissions Forum

Get expert LSAT preparation and law school admissions advice from PowerScore Test Preparation.

 Administrator
PowerScore Staff
  • PowerScore Staff
  • Posts: 8916
  • Joined: Feb 02, 2011
|
#98124
Complete Question Explanation

The correct answer choice is (A).

Answer choice (A): This is the correct answer choice.

Answer choice (B):

Answer choice (C):

Answer choice (D):

Answer choice (E):

This explanation is still in progress. Please post any questions below!
User avatar
 lsatstudent2
  • Posts: 26
  • Joined: Jan 16, 2023
|
#102020
Hi,

Is there a quick way to answer this question?
User avatar
 Jeff Wren
PowerScore Staff
  • PowerScore Staff
  • Posts: 389
  • Joined: Oct 19, 2022
|
#102066
Hello,

The first step to answering this question quickly is to have the proper setup, ideally with templates showing diagrams for when M is earlier than P (M - P) and when M is later than P (P - M). Since there are no ties in this game, those two scenarios show two major options or branches in the game. In other words, you will always be in one of those two situations.

Here is a link to the setup for your reference.

viewtopic.php?f=1489&t=36651

As for the question, it is a global Cannot be True question. A quick look at the answers shows that each answer lists a sequence of three variables (such as M - K - J for answer A).

Given that we have a conditional sequencing rule in the game (Rule 4), a good (i.e. strategic) place to start testing the answers would be to look for an answer that contains the variables in the necessary condition of Rule 4 (M - J - K) in a different order than given in that rule.

The reason that this would be a good place to start is that the way conditional rules work is that if the sufficient condition occurs, then (according to the rule) the necessary MUST also occur. Therefore, if we could find an answer where the sufficient occurs (M - P) but the necessary does NOT occur (meaning that we DON'T get M - J - K), then we have our answer.

To be clear, the test makers don't have to test this rule in this question, but there is a good chance that they will, as it is arguably the most difficult rule in the game. When you see that there are a couple of answers with M, J, K in different orders, that is a good sign to start with those answers.

As for Answer A (M - K - J), this is the correct answer as it violates Rule 4 (when combined with the other rules). We know from Rules 1 and 2 that J is always earlier than H and H is always earlier than P (J - H - P). If M is before J (as it is in this answer), then it would have to also be earlier than P. However if M is before P, Rule 4 gets triggered which would force the sequence of (M - J - K) in direct contradiction of this answer, so this sequence cannot happen based on the rules.

Answer B also has the M, J, K variables in a different order, specifically (K - J - M). However, unlike Answer A, this answer has M AFTER J (J - M), which would mean that we are in Template 2 and that M is also after P (P - M). Remember, there are no situations in the game where M is after J but before P. Either M is before P, which triggers Rule 4 and therefore M is also before J (Template 1) or M is after P.

Since P must be before M in this answer, Rule 4 does not get triggered and the (M - J - K) sequence is not required. In Template 2, it is perfectly fine to have K before J, so this answer choice (K - J - M) Could be True.

Get the most out of your LSAT Prep Plus subscription.

Analyze and track your performance with our Testing and Analytics Package.